Next: Solution to Problem 3.
Up: Solutions to Final Exam
Previous: Solution to Exercise 1.
Solution to Problem 2.
(2) [15 points]
Let
and
be i.i.d. uniform(0,1) rv's and
,
.
Show that the conditional distribution of
given
is uniform(0,
).
Solution:
It is fairly easy to show that the joint pdf of
is
and the marginal pdf for
is
Thus, the conditional pdf for
given
is
As a function of
, we recognize this as the pdf for
the uniform(
) distribution.
Dennis Cox
2003-01-18